The fence posts need to be painted Each post is round with a diameter of 50mm and length of 2.5m. The posts are hollow (no top or bottom) Now work out the surface area of a single fence post

Answers

Answer 1

Answer:

0.3927 m²

Step-by-step explanation:

Since a post is open at both ends, then it means it has no top nor bottom and as such the surface area is;

S.A = 2πrh

Where;

r is radius

h is height

We are given;

diameter; d = 50mm = 0.05 m

We know that; radius = diameter/2 = 0.05/2 = 0.025 m

Height; h = 2.5 m

Thus;

S.A = 2 × π × 0.025 × 2.5

S.A = 0.3927 m²


Related Questions

F(x) = 3x + 2 what if f(5)

Answers

Ans is 17 . Solved it Down below. Hope it help

ILL MARK BRAINIEST IF YOU DO THIS CORRECTLY!!!

Answers

Hmmm im on the same question so i think its this

What is the volume of the cylinder?
•576 cm3
•2887 cm3
•96 cm3
•192 cm3

Answers

[tex] \large\begin{gathered} {\underline{\boxed{ \rm {\red{Volume \: \: of \: \: cylinder \: = \: \pi \: {r}^{2} \: h }}}}}\end{gathered}[/tex]

r represents radius of cylinder.

h represents height of cylinder.

So ,

r = 6 cm

h = 16 cm

π = 3.14

Substuting the values

⇥Volume of the cylinder = π r² h

⇥Volume of the cylinder = 3.14 × 36 × 16

⇥Volume of the cylinder = 113.04 × 16

⇥Volume of the cylinder = 1808.64

Hence , the volume of cylinder is 1808.64 cm²

can someone help me with this one ....​

Answers

Answer:

-5, - 2, 3

Step-by-step explanation:

y=2x+3, y=-7, x=-5; y=-1, x=-2, y=9, x=3

Based only on the information given in the diagram, which congruence
theorems or postulates could be given as reasons why AABC= ALMN?
Check all that apply
O A. LL
O B. ASA
I C. LA
D. HL
E AAS

Answers

3 Answers:

Choice A.  LLChoice D.  HLChoice F.  SAS

==========================================================

Explanation:

Let's go through the answer choices one by one.

A) This can be used because LL = leg leg, and this means we have two pairs of congruent legs. Those pairs are AC = LN and CB = NM. The LL theorem only applies to right triangles.B) This cannot be used. We don't have info about two pairs of angles. We only know that one pair of angles are the same (those 90 degree angles). So we can't form the second "A" in "ASA". This idea will come up again in choice C and choice E.C) This cannot be used. Why not? Because the "A" of "LA" refers to "acute angle". But unfortunately we don't know anything about the acute angles (whether they are congruent or not). The LA theorem can only be applied to right triangles.D) This can be used. We can use the HL (hypotenuse leg) theorem because we see that AB = LM are the pair of congruent hypotenuses, and you can use any of the congruent leg pairs to form the L of HL. Similar to LL and LA, the HL theorem only works for right triangles.E) This cannot be used. Like with choice B, we can't form the second "A" of "AAS".F) This can be used because we have two pairs of congruent sides, with a pair of congruent angles between those sides. Those angles being the marked 90 degree angles. It turns out that LL theorem is a special case of the SAS theorem.

In short, we can use choice A, choice D, choice F. We can't use the other three choices because we lack the info about any other pairs of angles.

The congruence theorem or postulate that we can use to show that triangle ABC is congruent to triangle LMN is LL (Side-Side-Side), the correct option is A.

What are congruent triangles?

Suppose it is given that two triangles ΔABC ≅ ΔDEF

Then that means ΔABC and ΔDEF are congruent. Congruent triangles are exact same triangles, but they might be placed at different positions.

The order in which the congruency is written matters.

For ΔABC ≅ ΔDEF, we have all of their corresponding elements like angle and sides congruent.

Thus, we get:

[tex]\rm m\angle A = m\angle D \: or \: \: \angle A \cong \angle D \angle B = \angle E\\\\\rm m\angle B = m\angle E \: or \: \: \angle B \cong \angle E \\\\\rm m\angle C = m\angle F \: or \: \: \angle C \cong \angle F \\\\\rm |AB| = |DE| \: \: or \: \: AB \cong DE\\\\\rm |AC| = |DF| \: \: or \: \: AC \cong DF\\\\\rm |BC| = |EF| \: \: or \: \: BC \cong EF[/tex]

(|AB| denotes length of line segment AB, and so on for others).

We are given that;

Sides are equal

Now,

Based only on the information given in the diagram, we can use the following congruence theorems or postulates to show that triangle ABC is congruent to triangle LMN:

A. LL (Side-Side-Side): This theorem states that if three sides of one triangle are congruent to three sides of another triangle, then the triangles are congruent. In this case, we know that AB = LM, AC = LN, and BC = MN, so we can use LL to show that triangle ABC is congruent to triangle LMN.

B. ASA (Angle-Side-Angle): This theorem states that if two angles and the included side of one triangle are congruent to two angles and the included side of another triangle, then the triangles are congruent. In this case, we do not know any angle measures, so we cannot use ASA to show that the triangles are congruent.

Therefore, by the  congruent triangles the answer will be LL (Side-Side-Side).

Learn more about congruent triangles here:

https://brainly.com/question/16921692

#SPJ7

(4a)^2 without the exponents

Answers

Answer:

16 a*a

Step-by-step explanation:

(4a)^2

(4a) (4a)

16 a*a

Answer:

16a²

Step-by-step explanation:

(4a)²

=>(4)² x (a)²

=> 16 x a²

=>16a²

quadrilateral abcd is symmetric with respect to the y axis. the coordinates of point a are (-2,2), and the coordinates of point c are (2, 1). if b is in the first quadrant, what are the coordinates of b?​

Answers

Answer:

The coordinate of b = (2, 2)

Step-by-step explanation:

The details of the quadrilateral abcd are;

The quadrilateral is symmetric about the y-axis (the line x = 0)

The coordinates of the vertices are; a(-2, 2), c(2, 1)

The location of the vertex, b = The first quadrant

We have;

The line [tex]\overline {ab}[/tex] is perpendicular to the line [tex]\overline {bc}[/tex]

Let (x, y) represent the coordinate of the vertex b, we have;

(y - 2)/(x - (-2)) = (y - 2)/(x + 2) = -1/(y - 1)/(x - 2) = - (x - 2)/(y - 1)

(y - 2)·(y - 1) = -(x + 2)·(x - 2)

y² + x² - 3·y - 2 = 0...(1)

y² + x² = 3·y + 2

Also we have;

(y - 2)² + (x - 1)² + (y - 2)² + (x - (-2))² = (2 - 1)² + (2 - (-2))² = 17

Therefore;

2·y² + 2·x² - 8·y + 2·x + 13 = 17

2·(3·y + 2) - 8·y + 2·x + 13 = 17

Using an online tool, we have;

x = y

From equation (1), we have;

2·y² - 3·y - 2 = 0

∴ y = 2, or y = -1/2

Where y = 2, we have;

x = y = 2

Therefore, the point b = (2, 2).

Problem 3 Find the value of x.​

Answers

value of x should be 3.2 units

Answered by Gauthmath must click thanks and mark brainliest

3.2 units in your answer

Find the surface area of each figure. Round your answers to the nearest tenth, if necessary​

Answers

Answer: 109m^2

Step-by-step explanation:

(5)(8.4)/2 = 21 m^2

(4 ) (21) = 84 m^2

Base = 5^2 = 25 m^2

84 + 25 = 109 m^2

what is the gcf of 36, 126, and 210?

Answers

Answer:

Greatest common factor (GCF) of 36 and 210 is 6.

Step-by-step explanation:

Answer:

6

Brainliest, please! (Almost an Ace!)

Step-by-step explanation:

Look at the smallest factor, 36. Find its factors.

36: 1, 2, 3, 6, 12, 18, 36

Out of all of them, which is the biggest one that 126 and 210 are also divisible by?

We see that they're divisible by 1, 2, 3, and 6.

Our answer is 6.

A. 4√29/ 7 feet
b. 4√5 feet
c.√164 feet
d. 2√41

Answers

Answer:

These two triangles are similar triangles. This means that their side lengths are proportional to each other.

Thus, making line segment EC equal to "x", and BC equal to "y" we can write:

8/y = 28/(10+y)

The next step is to get rid of the fractions, which can be done by cross multiplying.

So we have:

8(10+y) = 28(y)

After distribution and some simplification, you should get the value of y.

80+8y = 28y

80 = 20y

80/20 = 20y/20

4 = y

y = 4

Knowing that y = BC, and y = 4, it is clear that BC = 4.

Since BC = 4, one can use the Pythagorean Theorem to solve for segment EC.

Pythagorean Theorem: a^2 + b^2 = c^2, where a and b are the side lengths of a right triangle, and c is the hypotenuse (in other words the longest side)

In our case, a and  b are 8 and 4 (the order doesn't really matter here).

So we have: 8^2 + 4^2 = c^2

64 + 16 = c^2

80 = c^2

c = sqrt 80

c  = 4 sqrt 5

And we arrive at the answer- EC = 4 sqrt 5, making B the correct choice.

Hope this helps!

Please help me find which expression is correct

Answers

Answer:

IN MY OPINION D NO IS THE CORRECT ANSWER OF YOUR QUESTION.

Answer:

in my opinion d is the correct answer of your questions.

Step-by-step explanation:

Because subtract subtract sign is always add.

hope this will help you

thanks

Find the least number by which the following number must be multiplied so that the product are perfect cube one number 72 to number 128 number 288 phone number 675​

Answers

Answer:

Well this question is actually a piece of cake. Just pick your favorite number. Multiply it by 10. Then do whatever operation you want with the 2,300. For the exponent part of this. Lets say we do it this way y times z equals 2,300. Exponents are letters used in mathematical terms. So any letter can be used to represent any number.

Step-by-step explanation:

I have 5 digits.My eight is worse 8000. One of my sixes is worth 60.The other is worth 10 times as much.My other digit is a zero

Answers

Answer:

i think its 58,660

Step-by-step explanation:

Answer:

58,660

Step-by-step explanation:

Plz help guys, it is a pretty easy sum​

Answers

Answer:

a3+13a

Step-by-step explanation:

sorry I don't know how to solve this

Answer:+-3

Explanation:

I added 4 on both side so that a^2+1/a^2-2 becomes a^2+1/a^2+2 which is Special products of (a^2+1/a^2).

please help in indices
[tex] \frac{ {5}^{m + 2} - {5}^{m} }{ {5}^{m + 1} + {5}^{m} } \\ \\ \frac{ {4}^{m} + {4}^{m + 1} }{ {4}^{m + 2} - {4}^{m} } [/tex]

Answers

Step-by-step explanation:

Hey there!

Please see your required answer in picture.

Hope it helps!

Please find attached photograph for your answer.

Hope it helps.

Do comment if you have any query.

Plz solve question 12

Answers

Answer: C

Step-by-step explanation:

To solve for 12, we can use eliminate or substituion to solve our system of equations. Let's use elimination method.

[tex]\left \{ {{3f-2k=10} \atop {-3f-2k=14}} \right.[/tex]

Let's add the equations together. This way, 3f+(-3f)=0

[tex]-4k=24[/tex]                   [divide both sides by -4]

[tex]k=-6[/tex]

Now that we know k, we can plug it into either equation to find f.

[tex]3f-2(-6)=10[/tex]        [multiply]

[tex]3f+12=10[/tex]             [subtract both sides by 12]

[tex]3f=-2[/tex]                    [divide both sides by 3]

[tex]f=-\frac{2}{3}[/tex]

Now that we have f and k, we know that C is the correct answer.

Explain the process due to which rain falls ? Class 4 - EVS​

Answers

Answer:

As you may already know, water drops that fall from the cloud are considered "rain".

The Sun's heat turns the moisture or water from leaves, plants, rivers, lakes, and oceans - and turns it into gas or also called, vapor. This water vapor then turns into gas and disappears into the air. When it gets mixed with the air, it cools down. When it cools down, it changes into small water drops, which then form a cloud. These small water drops join together with other water drops to create larger and bigger water drops.

You may know this now because this is the easiest part. What happens when something gets heavy or is over-filled? It falls down, right?

So, the large drops from water fall down as they get too heavy for the cloud to carry. These big droplets falling down on us are called Rain.

Which is the best estimate of -14 1/9 (-2 9/10)

Answers

Answer:

Step-by-step explanation:

-14 1/9 is close to - 14

-2 and 9/10 is close to - 3

The best estimate would be 42 (14 * 3)

Estimate means you put your calculator on the kitchen counter until you've done this question.

Let's see what the actual answer is. 40.92222 which is close to 41.

42 is a pretty good estimate.

Please help me with this anyone

Answers

Answer:

Step-by-step explanation:

Begin by combining like terms and then factoring. Combining like terms will give you

[tex]10p^2-17p-20=0[/tex] Using the "old-fashioned" way of factoring, the a times c method, our a = 10, b = -17 and c = -20.

a * c = 10(-20) = -200 and now we need the factors of 200 (don't worry about the negative) that combine to give us that middle term, -17p (here is where the negative matters). The factors of 200 are:

1. 200;  2, 100;  4. 50;  5, 40;  8, 25;  10, 20

The combination of those numbers that can be manipulated to give us a -17p is the 8, 25 as long as we say that the 25 is negative and the 8 is positive. Rewrite the original polynomial to reflect those factors:

[tex]10p^2-25p+8p-20=0[/tex] and then factor by grouping:

[tex](10p^2-25p)+(8p-20)=0[/tex] and factor out from each set of parenthesis what is common:

[tex]5p(2p-5)+4(2p-5)=0[/tex] again factor out what is common:

(2p - 5)(5p+ 4) = 0. These are the factors; therefore the solutions are

2p - 5 = 0 so

2p = 5 and

p = 5/2  and

5p + 4 = 0 and

5p = -4 so

p = -4/5

Jimmy is saving money to buy a concert ticket for $155. He has $42 so far and he can save $20 per week. In how many weeks will he have enough money to buy the ticket? Select an equation that could be used to answer the question above. Let W represent the number of weeks. A. 20w + 42 = 155 B. 20w - 155 =42 C. 42w + 20 = 155 D. 155w - 20 = 42

Answers

Answer:

The answer for this question is A

write as a sentence in words 7.8 > 3.4

Answers

Answer:

seven and eight tenths is greater than three and four tenths

Hope it helped you

Mohamad bought a remote control car and paid $70.20. The price before tax was $65.00. What percent sales tax did he pay?

Answers

Answer:

Mohamad payed 8% of sales tax.

Step-by-step explanation:

First, we need to find the additional amount he paid to find the tax percentage.

70.20 - 65.00 = $5.20

Now, we divided that answer by the original price to find the actual percentage.

5.20/65.00 = 0.08 = 8%

9) Assume that the random variable X is normally distributed, with mean = 90 and standard deviation o = 12. Compute the probability P(57 < X < 105).
A) 0.7888 B) 0.8944 C) 0.8914 D) 0.8819​

Answers

Answer:

Step-by-step explanation:

The mean, [tex]\bar{x}[/tex], is 90 and the standard deviation, [tex]\sigma[/tex], is 12.  We are looking for the probability that the variable X will fall between 57 and 105. We use the z-score table for this, AFTER we find the z scores. The formula to find the z-scores for us is:

[tex]P(\frac{57-\bar{x}}{\sigma}\leq z\leq \frac{105-\bar{x}}{\sigma})[/tex] and we fill in accordingly:

[tex]P(\frac{57-90}{12}\leq z\leq \frac{105-90}{12})[/tex] which simplifies to

[tex]P(-2.75\leq z\leq 1.25)[/tex] and we will break them up into 2 different sets as follows:

P(-2.75 ≤ z ≤ 0) + P(0 ≤ z ≤ 1.25)

and based on the fact that z scores are given from 0 on up, we are going to convert the first one by using the logic that if z is greater than -2.75 but less than 0, by symmetry, z is greater than 0 but less than 2.75:

P(0 ≤ z ≤ 2.75) + P(0 ≤ z ≤ 1.25) and we go to the z-score table.

Locate 2.7 down along the left side and move over til you're under the .05; that gives us the z-score for 2.75 which is .4970. Do the same for 1.25 to get a z-score of .3944. Add them together to get a final z-score that covers the range of values for X:

.4970 + .3944 = 0.8914

Pls help me I will mark your answer as brainliest!

Answers

Answer:

9

Step-by-step explanation:

if we divide it my 9 we will get 927,998.666667

Answer: greatest number which should be replace m= 8

Except number 9, ; 2,5.and 8 can replace m so that the number 5567m92 is divisible by 6( Here greatest among all is 8 so 8 is your answer)

Step-by-step explanation:

5567292 is divisible by 6.

5567592 is divisible by 6.

5567892 is divisible by 6.

Will Mark Brainlest Helppp please​

Answers

Answer:

1

Step-by-step explanation:

a pair of coordinates defines a first number x, and as second number y (or the functional result).

so, we need to find the value of x that leads to 2 as functional result.

2 = x² + 1

1 = x²

x = 1

SHORT ANSWER
Question 3
This expression has three restrictions. What are they?
(3x^2-3x/x-2)/(2x-10/x-4)
Answer in a complete sentence.

Answers

Answer:

The restrictions are x≠2,4,5

Step-by-step explanation:

First the denominators cannot be zero

x-2 ≠0  so x≠2

Then x-4 ≠0  so x≠4

Also, since we are dividing, when we flip the second expression, the numerator becomes the denominator, so it cannot be zero

2x-10≠0

2x≠10

x≠5

The restrictions are x≠2,4,5


Please help out explanation need it will

Answers

You just have to divide all of it and then round it to the nearest tenths if u need too .

Answer:

A=2(wl+hl+hw)

A=2(6ft×7ft+6ft×7ft+6ft×6ft)

A=2(42ft²+42ft²+36ft²)

A=2(120ft²)

A=240ft²

Step-by-step explanation:

can somebody please help meee??

Answers

Answer: d. He multiplied when he was supposed to divide

Step-by-step explanation:

He just multiplied across when he was supposed to divide (or cross multiply) to find the unit rate :)

6 less than twice a number

Answers

Answer:

6-2x

Step-by-step explanation:

six less than (subtraction) twice a number. 2 times a number x

Other Questions
6 points about carnivorous The computer technician charges $20 per visit plus $35 per hour. If the computer technician works for 2 hours, what will be the cost?$75$90$55 16. The ape . from branch to branch in search of food.1) lopes 2) swings 3) swoops 4) leaps solve this please sirplease pls help Your lifestyle desires should be a big part of choosing your career.Is this statement true or false?truefalse Blue Spruce University sells 4,500 season basketball tickets at $140 each for its 12-game home schedule. Give the entry to record (a) the sale of the season tickets and (b) the revenue recognized after playing the first home game. Six men working 8 hours a day dig a shamba in 5 days. How many more days will it take 4 men working 8 hours a day to finish the job?NOTE:PLEASE EXPLAIN IN DETAIL !!! Which number line shows the solution to-3-(-1) Doogan Corporation makes a product with the following standard costs: Standard Quantity or Hours Standard Price or Rate Direct materials 2.0 grams $ 7.00 per gram Direct labor 0.6 hours $ 14.00 per hour Variable overhead 0.6 hours $ 6.00 per hour The company produced 4,600 units in January using 10,120 grams of direct material and 2,100 direct labor-hours. During the month, the company purchased 10,690 grams of the direct material at $7.20 per gram. The actual direct labor rate was $14.55 per hour and the actual variable overhead rate was $5.90 per hour. The company applies variable overhead on the basis of direct labor-hours. The direct materials purchases variance is computed when the materials are purchased. The materials quantity variance for January is: Drag the tiles to the correct boxes to complete the pairs.Given that x= 3 + 81 and y= 7 - 1 match the equivalent expressions.-15 + 1958 + 106-&411-29 - 531I. 2y-y50 ty23 - 3y When we are considering AU and are thinking about how the utility gained by the majority could outweigh the disutility suffered by the minority, we are talking about a problem that deals with justice and human rights. A. True B. False What was an immediate goal of the Ku Klux Klan?A. to break the power of the Republican PartyB. to drive African Americans from the SouthC. to enslave African Americans D. To prevent African-Americans from voting The cost of producing pens with the company logo printed on them consists of a onetime setup fee of $265.00 plus $0.95 for each pen produced. This cost can be calculated using the formula C=265.00+0.95p, where p represents the number of pens produced and C is the cost. Use the formula to calculate the cost of producing 2900 pens. Pick out the incorrect statement about the partition (a) the year 1947 was a year of unprecedented violence and trauma of displacement (b) according to the two-nation theory, india consisted of two people-hindus and Muslims (c) punjab and bengal were bifurcated according to religious majority (d) there was a plan for transfer of the people across the border after partition Solve the triangle. Round your answers to the nearest tenth.HELP PLEASE !!!!A. mB.mC.mD.m If you are dealt 4 cards from a shuffled deck of 52 cards, find the probability of getting 2 queens and 2 kings.The probability is ___.(Round to six decimal places as needed.) how many moles of H2 and N2 can be formed by the decomposition of 0.145 mol of ammonia, NH3 ? Explain which intermolecular force is primarily responsible for the unusually high boiling point of H2OH2O. Compare the strengths of intermolecular forces of attraction present in a sample of liquid water to the strengths of intermolecular forces of attraction present in liquid samples of H2SH2S, H2SeH2Se, and H2TeH2Te. Justify your answer using the information presented in both tables, and complete the statements below. A sample of 13 sheets of cardstock is randomly selected and the following thicknesses are measured in millimeters. Give a point estimate for the population standard deviation. Round your answer to three decimal places. 1.96,1.81,1.97,1.83,1.87,1.84,1.85,1.94,1.96,1.81,1.86,1.95,1.89 how to solve 1168 divided by 8